- PowerScore Staff
- Posts: 5972
- Joined: Mar 25, 2011
- Sat May 05, 2018 11:07 am
#45440
Complete Question Explanation
(The complete setup for this game can be found here: lsat/viewtopic.php?t=11812)
The correct answer choice is (D)
Answer choice (A) is incorrect because, among other reasons, SO must be in folder 1 or 4 according to the fifth rule.
Answer choice (B) is incorrect because FM or SO must be in folder 1 according to the fourth and fifth rules.
Answer choice (C) is incorrect because FM or SO must be in folder 1 according to the fourth and fifth rules.
Answer choice (D) is the correct answer choice, and it is the only answer that provides an order which aligns with one of our two templates (Template #2 in the setup).
Answer choice (E) is incorrect because according to the third rule, folder 3 and folder 4 must have different subjects.
(The complete setup for this game can be found here: lsat/viewtopic.php?t=11812)
The correct answer choice is (D)
Answer choice (A) is incorrect because, among other reasons, SO must be in folder 1 or 4 according to the fifth rule.
Answer choice (B) is incorrect because FM or SO must be in folder 1 according to the fourth and fifth rules.
Answer choice (C) is incorrect because FM or SO must be in folder 1 according to the fourth and fifth rules.
Answer choice (D) is the correct answer choice, and it is the only answer that provides an order which aligns with one of our two templates (Template #2 in the setup).
Answer choice (E) is incorrect because according to the third rule, folder 3 and folder 4 must have different subjects.
Dave Killoran
PowerScore Test Preparation
Follow me on X/Twitter at http://twitter.com/DaveKilloran
My LSAT Articles: http://blog.powerscore.com/lsat/author/dave-killoran
PowerScore Podcast: http://www.powerscore.com/lsat/podcast/
PowerScore Test Preparation
Follow me on X/Twitter at http://twitter.com/DaveKilloran
My LSAT Articles: http://blog.powerscore.com/lsat/author/dave-killoran
PowerScore Podcast: http://www.powerscore.com/lsat/podcast/